K
Khách

Hãy nhập câu hỏi của bạn vào đây, nếu là tài khoản VIP, bạn sẽ được ưu tiên trả lời.

26 tháng 5 2017

cái này là bổ đề tui c/m rùi mà =="

13 tháng 3 2020

Sửa đề: n \(\ge1\).

Với n =1, bất đẳng thức trở thành đẳng thức.

Với n =2, cần chứng minh: \(2\left(a_1^2+a_2^2\right)\ge\left(a_1+a_2\right)^2\Leftrightarrow\left(a_1-a_2\right)^2\ge0\) (đúng)

Giả sử nó đúng đến n = k, tức là ta có: \(k\left(a_1^2+a_2^2+...+a_k^2\right)\ge\left(a_1+a_2+...+a_k\right)^2\)

Hay là: \(\left(a_1^2+a_2^2+...+a_k^2\right)\ge\frac{\left(a_1+a_2+...+a_k\right)^2}{k}\)

Ta c/m nó đúng với n = k +1 or \(\left(k+1\right)\left(a_1^2+a_2^2+...+a_k^2+a_{k+1}^2\right)\ge\left(a_1+a_2+...+a_k+a_{k+1}\right)^2\)

Ta có: \(VT=\left(k+1\right)\left(a_1^2+a_2^2+...+a_k^2+a_{k+1}^2\right)\)

\(\ge\left(k+1\right)\left[\frac{\left(a_1+a_2+...+a_k\right)^2}{k}+\frac{a^2_{k+1}}{1}\right]\ge\frac{\left(k+1\right)\left(a_1+a_2+..+a_k+a_{k+1}\right)^2}{k+1}=VP\)

Vậy đpcm là đúng.

P/s: Chả biết đúng không, chưa check, đại khái hướng làm là dùng quy nạp.

15 tháng 3 2020

delllllllllll bt

6 tháng 3 2021

a) Đặt \(d=\left(a_1,a_2,...,a_n\right)\Rightarrow\left\{{}\begin{matrix}a_1=dx_1\\a_2=dx_2\\...\\a_n=dx_n\end{matrix}\right.\) (với \(\left(x_1,x_2,...,x_n\right)=1\)).

Ta có \(A_i=\dfrac{A}{a_i}=\dfrac{d^nx_1x_2...x_n}{dx_i}=d^{n-1}\dfrac{x_1x_2...x_n}{x_i}=d^{n-1}B_i\forall i\in\overline{1,n}\).

Từ đó \(\left[A_1,A_2,...,A_n\right]=d^{n-1}\left[B_1,B_2,...,B_n\right]\).

Mặt khác do \(\left(x_1,x_2,...,x_n\right)=1\Rightarrow\left[B_1,B_2,...B_n\right]=x_1x_2...x_n\).

Vậy \(\left(a_1,a_2,...,a_n\right)\left[A_1,A_2,...,A_n\right]=d.d^{n-1}x_1x_2...x_n=d^nx_1x_2...x_n=A\).

28 tháng 5 2018

ÁP DỤNG BĐT Cauchy ta có : 

\(\text{a}_1+\text{a}_2+...+\text{a}_n\ge n^n\sqrt{\text{a}_1.\text{a}_2....\text{a}_n}\)  (1) 

\(\frac{1}{\text{a}_1}+\frac{1}{\text{a}_2}+...+\frac{1}{\text{a}_n}\ge n^n\sqrt{\frac{1}{\text{a}_1}\cdot\frac{1}{\text{a}_2}\cdot...\cdot\frac{1}{\text{a}_n}}\)(2) 

Nhân (1) và (2) vế với vế tương ứng ta có được BĐT (*) 

Đẳng thức xảy ra \(\Leftrightarrow\hept{\begin{cases}\text{a}_1=\text{a}_2=...=\text{a}_n\\\frac{1}{\text{a}_1}=\frac{1}{\text{a}_2}=...=\frac{1}{\text{a}_n}\end{cases}}\)

                             \(\Leftrightarrow\text{a}_1=\text{a}_2=...=\text{a}_n\)

NV
1 tháng 8 2020

Ta có:

\(1-a_1\ge a_2+a_3+...+a_n\ge\left(n-1\right)\sqrt[n-1]{a_2a_3...a_n}\)

\(1-a_2\ge a_1+a_3+...+a_n\ge\left(n-1\right)\sqrt[n-1]{a_1a_3...a_n}\)

....

\(1-a_n\ge a_1+a_2+...+a_{n-1}\ge\left(n-1\right)\sqrt[n-1]{a_1a_2...a_{n-1}}\)

Nhân vế với vế:

\(\left(1-a_1\right)\left(1-a_2\right)...\left(1-a_n\right)\ge\left(n-1\right)^n.a_1a_2...a_n\)

\(\Leftrightarrow\frac{a_1a_2...a_n}{\left(1-a_1\right)\left(1-a_2\right)...\left(1-a_n\right)}\le\frac{1}{\left(n-1\right)^n}\)

Dấu "=" xảy ra khi \(a_1=a_2=...=a_n=\frac{1}{n}\)